ms. simonaue and Mrs. law are racing in the big race. Their distance in meters and their time is measured in seconds. Ms. Simonaue ride is described in the equation.

[tex]y = \frac{3}{2} x + 4[/tex]
what is the description to match the equation. what is her rate? did she get a head start?​

Answers

Answer 1

Answer:

She started 4 meters ahead and moves forward 1.5 meters every second.

The rate is 1 and 1/2 meters per second and she did get a head start of 4 meters


Related Questions

help plz will give brainlyest

Answers

Answer:

A. Cendric is correct because he used the inverse of subtraction and added 4.5

Step-by-step explanation:

To solve for x, all we needed to do was to make x stand alone. To do this, we have to apply addition property of equality. This means we would add 4.5 to both sides for the equation to balance. Thus, we would have z standing alone which equals 3.

Therefore, Cendric was correct because he used the inverse of subtraction of -4.5, which is 4.5 that was later added to both sides of the equation.

Pls help will give brainlest to first person who answer correct 50 points

Answers

Answer:

-25/3

Step-by-step explanation:

Please let me know if you want me to add an explanation as to why this is the answer/how I got this answer. I can definitely do that, I just wouldn’t want to write it if you don’t want me to :)

Answer:

x = - 25/3

Step-by-step explanation:

10/3 = x/(-5/2)x = 10/3*(-5/2)x = - 25/3

What is the perimeter of the figure?

Answers

Answer:

138cm

Step-by-step explanation:

there are 2 sides equal to 27

27 + 27 = 54

there are 6 squares = to 14

6 × 14 = 84

54 + 84 = 138

Peyton and her children went into a restaurant and where they sell drinks for $3 each
and tacos for $4 each. Peyton has $40 to spend and must buy at least 10 drinks and
tacos altogether. If Peyton decided to buy 4 drinks, determine all possible values for
the number of tacos that she could buy. Your answer should be a comma separated
list of values. If there are no possible solutions, submit an empty answer.

Answers

Answer:

7,6

Step-by-step explanation:

So in total Peyton must buy 10 items. Since she's already buying 4 drinks. We need to find the amount of tacos the max amount of tacos she can buy are 7 tacos and the minimum is 6 because if we bought less than 6 it wouldn't have met the criteria for 10 items minimum. And if we passed 7 tacos it would go past the limit of how much money Peyton has. I hope this helped:)

Pls help I’m failing math I’ll brainlest

Answers

Answer:

z=-9 - 5.4

hope it helps

Step-by-step explanation:

please mark me brainliest

The cylinder has a surface area of 972cm. Find x. Round to the nearest whole number.
diameter: 2x
height: 5x

Answers

Answer:

A= 2pirh +2pir^2

972=2 x pi x x x 5x + 2 x pi x x^2

972= 31.4x^2 + 6.28x^2

972=37.68x^2

x^2= 972/37.68

x^2=25.796

x= 25.796 square root

x= 5

The value of x is 5 cm.

What is a cylinder?

A cylinder is a three-dimensional figure that has a radius and a height.

The volume of a cylinder is given as

Example:

The volume of a cup with a height of 5 cm and a radius of 2 cm is

Volume = 3.14 x 2 x 2 x 5 = 62.8 cubic cm

We have,

The surface area of a cylinder = 972 cm²

The surface area of a cylinder.

= 2πrh + 2πr²

Diameter = 2x

Radius = x

Height = 5x

Now,

2πrh + 2πr² = 972

2π (rh + r²) = 972

2π (5x² + x²) = 972

2π x 6x² = 972

6x² = 972/(2 x 3.14)

6x² = 154.78

x² = 25.79

x = √25.79

x = 5.08

x = 5

Thus,

x value is 5.

Learn more about cylinder here:

https://brainly.com/question/15891031

#SPJ2

Which fraction is equal to 35%?
O A.
100
350
O B.
100
35
C.
3.5
100
D.
35
100

Answers

Answer: D 35/100

Step-by-step explanation: if you divide 35/100, the answer would be .35, which is the decimal form of 35%.

The answer is d I think

If mQOS = 46, and mPOR = 61
and mPOQ = 28, what is mROS?

Answers

Answer:

ROS = 225°

Step-by-step explanation:

The four corners of the squares add up to 360°

Hope it helps you

Angles at a point add up to [tex]360^o[/tex]. The measure of [tex]m\angle ROS[/tex] is [tex]225^o[/tex]

Given that:

[tex]m\angle QOS = 46^o[/tex]

[tex]m\angle POR = 61^o[/tex]

[tex]m\angle POQ = 28^o[/tex]

The 4 angles (i.e. QOS, POR, POQ and ROS) are all angles at a point (i.e. point O).

This means that we can apply the angle at a point theorem.

The theorem is represented as:

[tex]m\angle QOS +m\angle POR +m\angle POQ + m\angle ROS = 360^o[/tex] --- i.e. angles at a point add up to [tex]360^o[/tex]

So, we have:

[tex]46^o + 61^o + 28^o + m\angle ROS = 360^o[/tex]

[tex]135^o + m\angle ROS = 360^o[/tex]

Collect like terms

[tex]m\angle ROS = 360^o -135^o[/tex]

[tex]m\angle ROS = 225^o[/tex]

Hence, the measure of [tex]m\angle ROS[/tex] is [tex]225^o[/tex]

Learn more about angles at:

https://brainly.com/question/15767203

Determine the relationship between the two triangles and whether or not they can be proven to be congruent

Answers

Answer:

The two triangles are related by AAS, so the triangles are congruent.

Step-by-step explanation:

Two angles and a non-included side of one triangle are congruent to corresponding two angles and an included side in the other triangle. Therefore, we can conclude that the two triangles are related by the AAS Congruence Criterion. Hence, both triangles congruent to each other.

Which relation is a function?


{(1, −1), (−2, 2), (−1, 2), (1, −2)}

{(1, 2), (2, 3), (3, 2), (2, 1)}​

{(1, 4), (2, 3), (3, 2), (4, 1)}

{(4, 2), (3, 3), (2, 4), (3, 2)}

Answers

Answer:

c

Step-by-step explanation:

because all the x values in c are different

the answer is c because it’s the only option with all differing x values

A cell phone company offers two plans to its subscribers. At the time new subscribers sign up, they are asked to provide some demographic information. The mean yearly income for a sample of 39 subscribers to Plan A is $55,575 with a standard deviation of $8,970. For a sample of 29 subscribers to Plan B, the mean income is $59,475 with a standard deviation of $6,942.
At the .025 significance level, is it reasonable to conclude the mean income of those selecting Plan B is larger? Hint: For the calculations, assume the Plan A as the first sample.
The test statistic is ______. (Negative amount should be indicated by a minus sign. Round your answer to 2 decimal places.)
The decision is _______
the null hypothesis that the mean of Plan B is larger.
The p-value is ______
(Round your answer to 2 decimal places.)

Answers

Answer:HI

Step-by-step explanation:HI

را آزرا
What is the value of x in the proportion?

Answers

Answer:

Step-by-step explanation:

Cross multiply on both sides

= (x + 1) (21) = 15(x + 3)

= 21x + 21 = 15x + 45

Bringing like terms on one side

21x - 15x = 45 - 21

= 6x = 24

x = 24/6 = 4

Option A is the correct answer

The correct option :

[tex] =\tt (a) \: 4[/tex]

Steps to derive correct option :

[tex] = \frac{x + 1}{x + 3} = \frac{15}{21} [/tex]

[tex] =( x + 1 )\times 21 = (x + 3 )\times 15[/tex]

[tex] = 21x + 21 = 15x + 45[/tex]

[tex] = 21x + 21 - 15x = 45[/tex]

[tex] = 6x + 21 = 45[/tex]

[tex] = 6x = 45 - 21[/tex]

[tex] = 6x = 24[/tex]

[tex] = x = \frac{24}{6} [/tex]

[tex] =\color{plum} \bold{x = 4}[/tex]

Let us now place 4 in the place of x and see if the substitution is equivalent to [tex] \frac{15}{21} [/tex] :

[tex] = \frac{4 + 1}{4 + 3} = \frac{15}{21} [/tex]

[tex] = \frac{5}{7} = \frac{15}{21} [/tex]

[tex] = \frac{5}{7} = \frac{15÷3}{21÷3} [/tex]

[tex] = \frac{5}{7} = \frac{5}{7} [/tex]

Therefore, the value of x in this proportion = 4

I really need help on this question

Answers

Answer:

[tex]r = 107 \\ q + s = 180 - 107 \\ = 73 \\ q = 73 \div 2 \\ = 36.5[/tex]

Tori earned $62.86 as a bonus from her job. She decided to buy as many CDs as possible since they were on sale for $8.80 each. Estimate how many CDs Tori can buy.

Answers

Answer:

approximately 7 CDs for her bonus

Answer:

Toris can buy about 7 CDs

Step-by-step explanation:

63 Divided by 9 = 7

4(x+4) + 2x = 52 need answer

Answers

Answer:

x=6

Step-by-step explanation:

Describe how to plot the point (-1, -2)

Where do you start? How many units do you go to the left or right? How many units it’s do you go up or down?

Answers

Answer:To plot the point you would start with -1 so you would go to the left till you reach -1 then to plot -2 you would go DOWN till you reach -2 Detail:the first number is ALWAYS how many times you go left or right the second number determine if you go up or down and negatives are always left and down and positive are always right and up unless you have 1,-5 then it’s to the right down and if you have -5, 7 you go left and up :3 hope this helps

Please help I’ll mark you as brainliest if correct

Answers

Answer:

72 cm³ (see below)

Step-by-step explanation:

First, refer to the volume formula:

V = l · w · h

If you plug in all of your values and simplify, you'll get the volume:

l = 6 cm

w = 3 cm

h = 4 cm

V = (6) (3) (4)

V = 18 (4)

V = 72 cm³

Because this is volume, the measurements are units cubed, meaning it's cm³.

Like he said 72cm3. Hope you get it right

Proportionally means
2 fractions (ratios) are NOT equal


2 fractions (ratios) are equal

Answers

Answer:

2 fractions are equal

Step-by-step explanation:

We say that two quantities are proportional if one is always the same multiple of the other.

Say a recipe calls for 2 cups of flour to 3 eggs. That proportion, 2 cups flour : 3 eggs, will always stay the same, no matter how many serving we make. If we make 2 servings, we'd double to 4 cups and 6 eggs, but that wouldn't change the proportion. After all,

[tex]\frac{2}{3}=\frac{4}{6}[/tex]

What number is 120% of 16?

Answers

Answer:19.2

Step-by-step explanation:

Gabe​ Amodeo, a nuclear​ physicist, needs 90 liters of a ​50% acid solution. He currently has a 30% solution and a 60% solution. How many liters of each does he need to make the needed 90 liters of ​50% acid​ solution?

Answers

Answer:

You need two equations: one for solution amounts and one for alcohol amounts.

Step-by-step explanation:

90 = X + Y

45 = (0.6)/X + (0.3)Y

Here X = amount of 60 % solution and Y = amount of 30 percent solution. Note that percentages have been converted to decimal amounts in the second equation.

From here you have to solve two equations with two unknowns. Easiest to solve first equation for X = 90 - Y, then substitute into second equation. this get you one equation in one unknown. Solving you you will get

X = 30 liters and Y = 60 liters.

will mark brainlyist

Answers

Answer:

x=18.7

Step-by-step explanation:

x+17+5x+10+4x-34=180

10x-7=180

10x=187

x=18.7

all angles in a trianlge add up to 180 degrees

Answer:

x = 17

Step-by-step explanation:

Since all angles of a triangle equal to 180 we can make this equation:

x + 17 + 6x + 10 + 4x - 34 = 180

11x - 7 = 180

11x = 187

x = 17

10 MINUTES UNTIL I HAVE TO TURN TEST IN
HELP PLS :)

Answers

Answer:

10) Total Discount = $15 dollars; New Amount = $60 dollars.

11) Total Markup = $3,750 dollars; New Amount = $28,750 dollars.

12a) Percent Increase from $100 to $150: 50% increase.

12b) Percent Decrease from $150 to $100: 33.33% decrease.

Step-by-step explanation:

I would be more than happy to show my work, but I see that you have to submit your test soon, so I will just leave it at this for now. If you need any clarification, I am more than happy to help. Feel free to give me Brainliest if you feel this helped! Have a good day, and good luck on your test. :)

What’s the function and domain and range. (1, -2) (-2, 0) (-1, 2) (1, 3)

Answers

Answer: Domain:

{

1

,

2

,

1

}

{

1

,

-

2

,

-

1

}

Range:

{

2

,

0

,

2

,

3

}

Step-by-step explanation: The domain is the set of all the values of

x

x

. The range is the set of all the values of

y

y

.

9-3 divided by 1/3 + 1

Answers

Answer:

1

Step-by-step explanation:

6(2x - 5) = -(x + 4)

Answers

[tex]6(2x - 5) = - (x + 4)[/tex]

[tex]12x - 30 = - x - 4[/tex]

[tex]13x = 26[/tex]

[tex]x = 2[/tex]

Answer:

x = 2

Step-by-step explanation:

Solve for x:

6(2x - 5) = -(x + 4)

Use the distributive property.

12x - 30 = -(x + 4)

12x - 30 = -x - 4

Add x to both sides.

13x - 30 = -4

Add 30 to both sides.

13x = 26

Divide both sides by x.

x = 2

Your final answer would be x = 2

Is the expression 3(x+1 1/2)-3 equivalent to 3x+ 1 1/2

Answers

Answer:

Yes

Step-by-step explanation:

3(x+5.50)-3

3x+16.50)-3

-3(3x+16.50)

(Click the Picture for my question) and thank you!

Answers

The answer should be C

Answer:

D

Step-by-step explanation:

By definition, π is defined to be the ratio of a circle's circumference to its diameter. In other words:

[tex]\displaystyle \pi=\frac{C}{d}[/tex]

In fact, if you multiply both sides by the diameter d, you can see that:

[tex]C=\pi d[/tex]

Which is the formula for circumference you've been commonly taught.

3/5 of the students in a classroom are girls. If there are ten boys in the class, how many total students are there?


15

25

16

20

Answers

Answer:

25

Step-by-step explanation:

15=Girls

10=Boys

25 =The total of the number of students

Someone please help me with this thank you

Answers

C because c is the right one

Please help!! Will make brainliest

Answers

UmmmmAnswer:

Step-by-step explanation:

Other Questions
Why is it possible to do the following party tricks without cheating?A. Stand with back and heels touching a wall and bend forward from the hips to touch the floor. Return to the upright position without flling or moving the root. B. Stand facing the edge of an open door with nose and abdomen touching the edge, the feetstraddling the door, and the hands to the sides. Attempt to rise up on the toes.Observe what body motions occur to keep the center of gravity within the base of support whenthese activities are performed without the impediments. Can some one help thanks An entrepreneur recently purchased Cocoon's, a local deli, on the beach. To operate the business, she estimates that selling and administrative expenses will be $98,510.00 per year and that depreciation will be $20,000.00 per year. As part of her purchase, she took out a bank loan that will require $76,265.00 per year in interest. She anticipates paying a 32.00% tax rate on income. She estimates that the deli will attract 200.00 customers per day. Each customer will spend $12.00 on average. The cost of goods per customer will be $4.50. She will operate the deli 350.00 days per year. What is the projected net profit margin for the deli Cmo fueron los primeros tangos?a. sin letrab. con letraC. con msicad sin msica Produced, growing, living, or occurring natively or naturally in a particular region or environment A small sphere of reference-grade iron with a specific heat of 447 J/kg K and a mass of 0.515 kg is suddenly immersed in a water-ice mixture. Fine thermocouple wires suspend the sphere, and the temperature is observed to change from 15 to 14C in 6.35 s. The experiment is repeated with a metallic sphere of the same diameter, but of unknown composition with a mass of 1.263 kg. If the same observed temperature change occurs in 4.59 s, what is the specific heat of the unknown material Any trace of an ancient organism preserved in rock is called a Which word is a synonym of influential?unmovingusefulsickpowerful Find the slope of a line perpendicular to y = -3/4x + 5. show your work. help pls i will give you brainlist but make sure its correct help ASAP 4 dogs have weights of 5.2 kg, 3.5 kg, 7 kg, and 9.74 kg. What is the total weight of the dogs? What is the average weight of the 4 dogs? Help me out with the answer Help asap will give brainliest What are the costs and sources of inefficiency in a barter B apply.) A. Productivity is increased by specialization. B. Each good has only one price. C. There is increased time and effort spent looking for trading partners. D. There is a lack of standardization. E. There is difficulty in accumulating wealth. OF. Transactions costs are almost always high. What is the name of the line passing through the vertex that dividesthe parabola into two symmetric parts? a knife is three times the cost of a spoon. 9 spoons and 12 knives cost 82.80. Work out the cost of one knife. Question / ofIf making inferences were to be represented by a tree, what would thebranches represent?A. ObservationsB. InferencesC. Inference cluesD. Notes An analysis of the company's insurance policies provided the following facts.Policy Date of Purchase Months of Coverage Cost A April 1, 2017 24 $10,824 B April 1, 2018 36 9,576 C August 1, 2019 12 8,424 The total premium for each policy was paid in full (for all months) at the purchase date, and the Prepaid Insurance account was debited for the full cost. (Year-end adjusting entries for Prepaid Insurance were properly recorded in all prior years.)Required:So what would my adjusting journal entry be? Timothy read the following article about golf for a research paper. Golf is such a unique and rewarding sport. Theres nothing like the sensation of hearing the club whack the golf ball into the air. And the perfectly manicured greens are so lush and inviting. Youre not required to sprint around and exhaust yourself physically. Undoubtedly everyone should give golf an opportunity. Analyze how the authors point of view affects the reader. A. The reader will think that golf courses are the most beautiful outdoor spaces in the world. B. The reader will refuse to play other outdoor sports because of a fear of exhaustion. C. Because the author thinks golf is a unique sport, the reader will select other golf-related stories to read. D. Because the author thinks golf is a unique sport, the reader will likely try golf or at least think positively about it. (6 1/2+2 2/5 )+ 2 1/2 using properties of addition